What are the changes in the mass number A and the atomic number Z of a nucleus that undergoes ß+ decay?a. A is unchanged, Z decreases by 2.b. A is unchanged, Z decreases by 1.c. A is unchanged, Z increases by 1.d. A decreases by 2, Z increases by 1.e. A decreases by 4, Z decreases by 2.

Answers

Answer 1

The changes in the mass number A and the atomic number Z of a nucleus that undergoes ß+ decay are b. A is unchanged, Z decreases by 1. ß+ decay, also known as positron emission, occurs when a nucleus loses a particle called a positron.

This particle has the same mass as an electron but has a positive charge. During ß+ decay, a proton in the nucleus is converted into a neutron, and a positron and a neutrino are emitted.

- Mass number (A): Represents the total number of protons and neutrons in the nucleus. Since a proton changes into a neutron, the total number of nucleons remains the same. Therefore, A is unchanged.
- Atomic number (Z): Represents the number of protons in the nucleus. Since a proton changes into a neutron during ß+ decay, the number of protons decreases by 1. Therefore, Z decreases by 1.


The mass number, A, of the nucleus remains unchanged during ß+ decay. This is because the number of protons and neutrons in the nucleus does not change. However, the atomic number, Z, decreases by 1. This is because the nucleus loses a positively charged proton, which is replaced by a neutrally charged neutron.

In ß+ decay, a nucleus undergoes a transformation in which a proton changes into a neutron, and a positron (ß+) is emitted.


Learn more about neutrino here:

brainly.com/question/13089783

#SPJ11


Related Questions

"As a charged particle movers freely in a circular path in the presence of a constant magnetic field applied perpendicular to the particles velocity, the particle's kinetic energy (a) remains constant (b) increases (c) decreases (19.4)"
(a) remains constant

Answers

As a charged particle moves freely in a circular path in the presence of a constant magnetic field applied perpendicular to the particle's velocity, the particle's kinetic energy (a) remains constant.

This is because the magnetic force acting on the charged particle is always perpendicular to its velocity, resulting in no work being done on the particle. Consequently, there is no change in its kinetic energy as it continues to move in a circular path.

The term “magnetic force” which can be explained as the force experienced by the electric charge, electric current and magnetic objects due to the magnetic field this force acts perpendicular to the direction of velocity of charge, current or magnetic material as in the above solution the moving charge travels experienced the magnetic force in negative Z-direction due to the magnetic field in which it was moving thus we can say that magnetic force is an vector quantity.

To learn more about magnetic field  https://brainly.com/question/14411049

#SPJ11

An object of mass m and moment of inertia I has rotational kinetic energy KR. Its angular momentum is:

Answers

The angular momentum of an object of mass m having a moment of inertia I and a rotational kinetic energy KR is L = I * √(2 * KR / I).

The rotational kinetic energy is given by the formula:

KR = (1/2) * I * ω²

Where ω is the angular velocity of the object in radians per second. The angular momentum (L) of the object is related to its moment of inertia and angular velocity by the formula:

L = I * ω

To find the angular momentum, we can rearrange the rotational kinetic energy formula to solve for ω:

ω = √(2 * KR / I)

Now, substituting this expression for ω in the angular momentum formula, we get:

L = I * √(2 * KR / I)

This equation expresses the angular momentum (L) of an object with mass m, moment of inertia I, and rotational kinetic energy KR. The angular momentum is a vector quantity that represents the rotational analog of linear momentum and depends on both the object's moment of inertia and its angular velocity.

Learn more about angular momentum here: https://brainly.com/question/30338110

#SPJ11

a space probe 2.0 * 1010 m from a star measures the total intensity of electromagnetic radiation from the star to be 5.0 * 103 w>m2 . if the star radiates uniformly in all directions, what is its total average power output?

Answers

The total average power output of the star is 2.63 * 10^26 watts.

To find the total average power output of the star, we need to use the formula for the surface area of a sphere and the inverse square law for radiation. The formula for the surface area of a sphere is 4πr^2, where r is the distance from the center of the sphere to its surface.

Using this formula, we can find the surface area of a sphere with a radius of 2.0 * 10^10 m:

[tex]Surface area = 4πr^2\\Surface area = 4π(2.0 * 10^10)^2\\Surface area = 5.02 * 10^21 m^2\\[/tex]

Next, we need to use the inverse square law for radiation, which states that the intensity of radiation decreases with the square of the distance from the source. In other words, if the distance from the source is doubled, the intensity of radiation decreases by a factor of four.

Using this law, we can find the power output of the star:

Power output = intensity * surface area / 4πr^2
Power output = (5.0 * 10^3 W/m^2) * (5.02 * 10^21 m^2) / (4π(2.0 * 10^10)^2)
Power output = 2.63 * 10^26 W

Therefore, the total average power output of the star is 2.63 * 10^26 watts.

To know more about power output click here:

https://brainly.com/question/13937812

#SPJ11

After landing on an unfamiliar planet, a space explorer constructs a simple pendulum of length 54.0 cm. The explorer finds that the pendulum completes 99.0 full swing cycles in a time of 139s. What is the magnitude of the gravitational acceleration on this planet?
g_planet = ___ m/s^2

Answers

The magnitude of the gravitational acceleration on the unfamiliar planet is approximately 2.83 m/s^2.To find the gravitational acceleration on the unfamiliar planet, we can use the formula:

g = (4π²L) / T²

where g is the gravitational acceleration, L is the length of the pendulum, and T is the time for one full swing cycle.

Substituting the given values, we get:

g = (4π² × 0.54 m) / (99.0 × 2 × π)^2 × 139^2 s^2

Simplifying this expression, we get:

g = 2.83 m/s^2

Therefore, the magnitude of the gravitational acceleration on the unfamiliar planet is approximately 2.83 m/s^2.
To calculate the gravitational acceleration (g_planet) on the unfamiliar planet using the simple pendulum, you can follow these steps:

1. Determine the period (T) of the pendulum, which is the time it takes to complete one full swing cycle.

T = total time / number of cycles = 139s / 99.0 cycles = 1.40404s

2. Use the formula for the period of a simple pendulum:

T = 2π * √(L / g_planet)

Where L is the length of the pendulum (0.54 m), and g_planet is the gravitational acceleration we want to find.

3. Rearrange the formula to solve for g_planet:

g_planet = (4π² * L) / T²

4. Plug in the values:

g_planet = (4π² * 0.54 m) / (1.40404s)²

5. Calculate the result:

g_planet ≈ 9.60 m/s²

So, the magnitude of the gravitational acceleration on this unfamiliar planet is approximately 9.60 m/s².

learn more about gravitational acceleration here; brainly.com/question/28188525

#SPJ11

Blackbody radiation can also be used to study stars. If Star A radiates light with a maximum intensity of 650 nm and Star B with a maximum intensity of 480 nm, which star appears more blue? The visible range of the electromagnetic spectrum is about 400-750 nm

Answers

If Star A radiates light with a maximum intensity of 650 nm and Star B with a maximum intensity of 480 nm,  Star B appears more blue than Star A.

Blackbody radiation refers to the radiation emitted by an object due to its temperature. The intensity of this radiation depends on the temperature and the wavelength of the emitted light. In the case of the two stars, Star A radiates light with a maximum intensity of 650 nm and Star B with a maximum intensity of 480 nm.

The visible range of the electromagnetic spectrum is between 400-750 nm. Since Star A has a maximum intensity at 650 nm, it means that it emits red light with a wavelength longer than 650 nm. On the other hand, Star B has a maximum intensity at 480 nm, which means that it emits blue light with a wavelength shorter than 480 nm.

Therefore, Star B appears more blue than Star A.

To learn more about black body radiation https://brainly.com/question/12531332

#SPJ11

calculate the magnitude of the acceleration, in meters per second squared, of a proton from rest in such an electric field.

Answers

The magnitude of the acceleration of a proton from rest in a 7.5 x [tex]10^{6[/tex] N/C electric field is approximately 7.19 x [tex]10^{14} m/s^2[/tex].

The acceleration of a proton in an electric field can be calculated using the formula:

a = F/m

where a is the acceleration of the proton, F is the electric force acting on the proton, and m is the mass of the proton.

The electric force acting on the proton can be calculated using the formula:

F = qE

where F is the electric force, q is the charge of the proton, and E is the electric field strength.

The charge of a proton is +1.6 x [tex]10^{-19[/tex] C, and the electric field strength is 7.5 x [tex]10^{6[/tex] N/C. Substituting these values into the formula for the electric force, we get:

F = (1.6 x [tex]10^{-19[/tex] C) x (7.5 x[tex]10^{6[/tex] N/C)

F = 1.2 x [tex]10^{-12[/tex] N

The mass of a proton is 1.67 x [tex]10^{-27[/tex] kg.

Substituting this value and the calculated force into the formula for acceleration, we get:

a = (1.2 x [tex]10^{-12[/tex] N) / (1.67 x [tex]10^{-27[/tex] kg)

a = 7.19 x [tex]10^{14} m/s^2[/tex]

For similar question on magnitude of the acceleration

https://brainly.com/question/29678420

#SPJ11

Question :-

Suppose there is a 7.5 x [tex]10^6[/tex] N/C electric field in some region. Calculate the magnitude of the acceleration, in meters per second squared, of a proton from rest in such an electric field.

Approximate percentage of electrical energy converted to heat in the average incandescent lightbulb.100%95%30%15%1%

Answers

The approximate percentage of electrical energy converted to heat in the average incandescent light bulb is 95%.

In the average incandescent light bulb, approximately 95% of the electrical energy is converted to heat, while only about 5% is converted to visible light. This is because incandescent light bulbs work by heating a filament until it becomes hot enough to emit visible light. However, as the filament heats up, it also radiates a significant amount of energy in the form of heat, which is not useful for lighting purposes.

This is one of the reasons why incandescent light bulbs are being phased out in many countries in favor of more energy-efficient alternatives such as LED (light-emitting diode) bulbs. LEDs convert a much higher percentage of the electrical energy into visible light, resulting in significant energy savings and reduced heat output.

To learn more about electrical energy visit: https://brainly.com/question/16182853

#SPJ11

A machine gear consists of 0.10 kg of iron and 0.16 kg of copper. How much total heat is generated in the part if its temperature increases by 35 C°? (Specific heats of iron and copper are 450 and 390 J/kg×°C, respectively.)

Answers

To calculate the total heat generated in the part, we need to use the formula:

Q = mcΔT

Where Q is the total heat generated, m is the mass of the part, c is the specific heat of the material, and ΔT is the change in temperature.

Given:

Mass of iron (m1) = 0.10 kg
Specific heat of iron (c1) = 450 J/kg°C
Mass of copper (m2) = 0.16 kg
Specific heat of copper (c2) = 390 J/kg°C
Change in temperature (ΔT) = 35°C

To find the total heat generated, we need to calculate the heat generated by each material and then add them together.

Heat generated by iron (Q1) = m1c1ΔT
= 0.10 kg x 450 J/kg°C x 35°C
= 1575 J

Heat generated by copper (Q2) = m2c2ΔT
= 0.16 kg x 390 J/kg°C x 35°C
= 2184 J

Total heat generated (Q) = Q1 + Q2
= 1575 J + 2184 J
= 3759 J

Therefore, the total heat generated in the part is 3759 J.

TO KNOW MORE ABOUT Heat genration CLICK THIS LINK-

brainly.com/question/21959031

#SPJ11

In a 60 Hz AC system, what is the duration in seconds for one complete cycle

Answers

In a 60 Hz AC system, the duration for one complete cycle is 1/60 seconds or 0.0167 seconds. This means that the alternating current changes direction or polarity 60 times in a second.


The concept of AC or alternating current is based on the idea of changing the direction of current flow in a circuit. The frequency of this change determines how many times the current changes direction in a second.

In a 60 Hz AC system, the current completes one full cycle from positive to negative and back to positive again in 1/60 seconds.

The significance of the frequency of AC power lies in the fact that it determines the speed of operation of electrical devices that use this power.

For example, if the frequency of AC power is changed from 60 Hz to 50 Hz, it can affect the performance of motors, transformers, and other devices that rely on the frequency for their operation.


In summary, the duration for one complete cycle in a 60 Hz AC system is 1/60 seconds or 0.0167 seconds.

This frequency is critical for the proper functioning of many electrical devices and systems, and any changes in it can have a significant impact on their performance.

To know more about alternating current here

https://brainly.com/question/11673552

#SPJ11

The bit-rate for a 16-bit stereo sound sampled at 32 kHz is 1.024 Mbps. What is the highest usable audible frequency in the digital media file?

Answers

The highest usable audible frequency in the digital media file is 16 kHz.

What is the highest usable audible frequency in the digital media file?

To find the highest usable audible frequency in a 16-bit stereo sound sampled at 32 kHz with a bit-rate of 1.024 Mbps, you can use the Nyquist theorem. The theorem states that the highest frequency that can be accurately represented is half of the sampling rate.

Identify the sampling rate.
The sampling rate is 32 kHz.

Apply the Nyquist theorem.
The highest usable audible frequency = (Sampling rate) / 2
The highest usable audible frequency = 32 kHz / 2

The digital media file's maximum usable audible frequency is 16 kHz.

Learn more about frequency.

brainly.com/question/5102661

#SPJ11

a moth at about eye level is 13.5 cm in front of a plane mirror; you are behind the moth, 33.8 cm from the mirror. what is the distance between your eyes and the apparent position of the moth's image in the mirror?

Answers

The distance between your eyes and the apparent position of the moth's image in the mirror is 47.3 cm.

To find the distance between your eyes and the apparent position of the moth's image in the mirror, we need to consider the distance of the moth from the mirror and your distance from the mirror.

The moth is 13.5 cm in front of the mirror. Since a plane mirror creates a virtual image at the same distance behind the mirror as the object is in front of the mirror, the moth's image will also be 13.5 cm behind the mirror.

You are 33.8 cm from the mirror. To find the total distance between your eyes and the moth's image, you need to add the distance from your eyes to the mirror (33.8 cm) and the distance of the moth's image behind the mirror (13.5 cm).

Distance between your eyes and moth's image = 33.8 cm + 13.5 cm = 47.3 cm

So, the apparent position of the moth's image in the mirror is 47.3 cm away from your eyes.

Learn more about "distance": https://brainly.com/question/26550516

#SPJ11

a spider sits on a turntable that is rotating at a constant 33 rpm. The velocity, v, of the spider is

Answers

The turntable is rotating at a constant 33 rpm, the spider's velocity will also be constant as long as it remains on the turntable. The velocity of the spider on the turntable is also constant.

Axial drift velocity is another name for it. At the Fermi velocity, an electron will typically go through a conductor at random. This random motion will experience a minor net flow velocity in one direction as a result of an applied electric field. However, it is important to note that the velocity is not the same as the speed of the spider. The velocity takes into account both the speed and direction of the spider's movement on the turntable.

velocity is equal to time/space.wavelength times frequency equals velocity.

These two equations can be used to determine velocity using various variables.

Learn more about Velocity here

https://brainly.com/question/17127206

#SPJ11

the force as a function of displacement of a moving object is presented by the graph. how much work is done when the object moves from 0 m to 5 m?

Answers

The work done by the force on the object as it moves from 0 m to 5 m is 100 Joules.

To calculate the work done when an object moves from 0 m to 5 m, we need to find the area under the graph of force as a function of displacement. The graph may be a straight line or a curve, but we can use the formula for calculating the area of a trapezoid to find the work done.

The formula for calculating the area of a trapezoid is: Area = 1/2 × (a + b) × h, where a and b are the parallel sides and h is the height.

In this case, the parallel sides are the force values at 0 m and 5 m, and the height is the displacement of the object. Let's assume that the force values at 0 m and 5 m are 10 N and 30 N respectively. Therefore, the area of the trapezoid is:

Area = 1/2 × (10 N + 30 N) × 5 m
Area = 100 J

So, the work done by the force on the object as it moves from 0 m to 5 m is 100 Joules. This means that the force applied on the object did 100 Joules of work, which is equal to the amount of energy transferred to the object as it moved from 0 m to 5 m.

For more such questions on Work done.

https://brainly.com/question/26047403#

#SPJ11

How does simulation synthesis differ from other synthesis methods?

Answers

Unlike other synthesis methods, such as deductive synthesis or inductive synthesis, simulation synthesis focuses on creating a realistic simulation of the system in question

Simulation synthesis is a type of synthesis method that involves creating virtual models of a system or process in order to analyze its behavior. This simulation is then used to test different scenarios and determine the best course of action. The key difference between simulation synthesis and other methods is that simulation synthesis allows for a more detailed and nuanced analysis of the system, taking into account factors that may not be immediately apparent or easily quantifiable.

Additionally, simulation synthesis can be used to test a wide range of scenarios and variables, providing a more comprehensive understanding of how the system operates. Overall, simulation synthesis offers a powerful tool for understanding and optimizing complex systems and processes.

More on synthesis: https://brainly.com/question/28980572

#SPJ11

consider an object containing 12 one-dimensional oscillators (this object could represent a model of 4 atoms in an einstein solid). there are 4 quanta of vibrational energy in the object. (a) how many microstates are there, all with the same energy?

Answers

The number of microstates with the same energy is 1365.

The number of one-dimensional oscillators, N = 12

The number of quanta, n = 4

The equation for calculating the number of microstates is given by,

Ω = (n + N- 1)! /n! (N - 1)!

Ω = (4 + 12 -1)! /4! (12 -1)!

Ω = 15!/(4! x 11!)

Ω = 1365

To learn more about microstates, click:

https://brainly.com/question/31493438

#SPJ4

if the coefficient of kinetic friction between tires and dry pavement is 0.800, what is the shortest distance in which an automobile can be stopped by locking the brakes when traveling at 26.3

Answers

When locking the brakes at 26.3 m/s, the shortest stopping distance is approximately 44.1 meters.

To find the shortest stopping distance, we'll use the following terms and steps:

1. Coefficient of kinetic friction (μk) = 0.800
2. Initial velocity (v₀) = 26.3 m/s
3. Final velocity (v) = 0 m/s (the car comes to a stop)
4. Acceleration (a) = -μk × g (g = 9.81 m/s², acceleration is negative as it's decelerating)

Calculate the acceleration.
a = -0.800 × 9.81
a = -7.848 m/s²

Use the equation v² = v₀² + 2 × a × d, where d is the stopping distance.

Rearrange the equation to solve for d.
d = (v² - v₀²) / (2 × a)

Substitute the values into the equation and solve for d.
d = (0² - (26.3)²) / (2 × -7.848)
d = (-691.69) / (-15.696)
d ≈ 44.1 meters

So, the shortest stopping distance when locking the brakes at 26.3 m/s is approximately 44.1 meters.

More on distance: https://brainly.com/question/14849740

#SPJ11

(C) Fg = Gm1m2/r2 and FE = kq1q2/r2

The nuclear force does not have a similar relationship
Forces between two objects which are inversely proportional to the square of the distance between the objects
include which of the following?

I. Gravitational force between two celestial bodies
II. Electrostatic force between two electrons
III. Nuclear force between two neutrons

(A) I only
(B) III only
(C) I and II only
(D) II and III only
(E) I, II, and III

Answers

The statement "Forces between two objects which are inversely proportional to the square of the distance between the objects" refers to two fundamental forces, namely the gravitational force and the electrostatic force. The correct option is C.

The electrostatic force is a fundamental force of nature that results from the interaction between electrically charged particles. It can either attract or repel charged particles depending on their charges. Electrostatic force plays a crucial role in many physical phenomena, including the behavior of atoms, molecules, and materials.

Option (B) is not true because the electrostatic force is a fundamental force that is inversely proportional to the square of the distance between two charged particles, but it does not involve neutrons.

Option (D) is not true because it includes the nuclear force between two neutrons, which is not inversely proportional to the square of the distance between the particles. The nuclear force is a short-range force that acts only between particles in the nucleus and has a much more complex relationship than the simple inverse square law.

Therefore, the correct answer is an option (C) I and II only.

To learn about Coulomb's law click:

brainly.com/question/506926

#SPJ4

A 28.7 kg sled is pulled forward with a 63.0 N force across ground with uk = 0.169. What is the acceleration of the sled?

Answers

The acceleration of the sled is 0.54 m/s², for a 28.7 kg sled is pulled forward with a 63.0 N force across the ground with uk = 0.169.

The force of friction acting on the sled can be calculated using the formula:

f_friction = uk × f_normal

where uk is the coefficient of kinetic friction and f_normal is the normal force acting on the sled.

f_normal = m × g

where m is the mass of the sled and g is the acceleration due to gravity (9.8 m/s²).

f_normal = 28.7 kg × 9.8 m/s² = 281.26 N

f_friction = 0.169 × 281.26 N = 47.50 N

The net force acting on the sled can be calculated using the formula:

f_net = f_applied - f_friction

where f_applied is the applied force on the sled.

f_applied = 63.0 N

f_net = 63.0 N - 47.50 N = 15.50 N

The acceleration of the sled can be calculated using the formula:

a = f_net / m

a = 15.50 N / 28.7 kg = 0.54 m/s²

Therefore, the acceleration of the sled is 0.54 m/s².

Learn more about acceleration at

https://brainly.com/question/12550364

#SPJ4

A positively charged rod is brought close to an uncharged electroscope. While the rod is close, one's finger touches the far side of the metal ball on the electroscope. The finger is removed and then the rod is taken away. The electroscope is

Answers

When a positively charged rod is brought close to an uncharged electroscope, the electroscope will become polarized. This means that the electrons in the electroscope will move away from the positively charged rod and towards the far end of the electroscope.

This results in a separation of charges within the electroscope, with the far end becoming negatively charged and the near end becoming positively charged.

When one's finger touches the far side of the metal ball on the electroscope, any excess charge on the electroscope is conducted away by the finger. This effectively neutralizes the electroscope, since any excess charge has been transferred to the person's body.

When the finger is removed and the rod is taken away, the electroscope is still neutral. This is because the charge on the rod did not have enough time to transfer to the electroscope before it was neutralized by the finger. Therefore, the electroscope remains in its original uncharged state.

In summary, bringing a positively charged rod close to an uncharged electroscope will cause polarization, touching the electroscope with a finger will neutralize any excess charge, and removing the rod will not leave the electroscope with a net charge.

For more such questions on Electroscope.

https://brainly.com/question/10470196#

#SPJ11

450 C of charge flows through a motor and 9000 J of energy are converted in the motor. 1800 J are dissipated in the cell. The EMF of the cell is:

Answers

The EMF of the cell is 24 volts.

We can use the relationship between charge, energy, and EMF to solve this problem. The energy converted in the motor is equal to the product of the EMF of the cell and the charge that flows through it:

E = EMF * Q

where E is the energy, EMF is the electromotive force of the cell, and Q is the charge.

From the problem statement, we know that 450 C of charge flows through the motor and 9000 J of energy are converted in the motor. We also know that 1800 J of energy are dissipated in the cell. Therefore, the total energy provided by the cell is:

E_total = E_motor + E_dissipated

= 9000 J + 1800 J

= 10800 J

Using the equation above, we can solve for the EMF of the cell:

EMF = E/Q = E_total/Q = 10800 J/450 C = 24 V

Therefore, the EMF of the cell is 24 volts.

To learn more about electromotive visit:

https://brainly.com/question/13753346

#SPJ11

What is the minimum voltage needed to generate active force in the skeletal muscle?

Answers

The minimum voltage needed to generate active force in the skeletal muscle is threshold voltage which is around -50 and -55 mV.

The skeletal muscle activation is dependent on voltage channels which allow change in sodium and potassium ion concentration on the nerve membrane. The neuromuscular junction allows the communication and the change in concentration of ions. The threshold voltage required to bring about the change is around -50 and -55 mV.

The action potential occurs in three steps, depolarization, repolarization and hyperpolarisation. Depolarisation spreads further lead to change in potential of sarcoplasmic reticulum which is the storehouse of calcium ions. These ions are released and bring about changes in protein structure making them available to bind to locomotory proteins.

Learn more about skeletal muscle -

https://brainly.com/question/31116738

#SPJ4

A charge of −5.2 × 10−6 C is placed at a point in space where the electric field is directed toward the right and has a magnitude of 6.4 × 105 N/C. What is the magnitude of the electrostatic force on the charge?

Answers

The magnitude of the electrostatic force on the charge is approximately 3.328 × 10^-6 N.

To find the magnitude of the electrostatic force on the charge, you can use the formula:

F = q * E

where F is the electrostatic force, q is the charge, and E is the electric field.

Given:
Charge (q) = -5.2 × 10^-6 C
Electric field (E) = 6.4 × 10^5 N/C

F = (-5.2 × 10^-6 C) * (6.4 × 10^5 N/C)
F ≈ -3.328 × 10^-6 N

Since we want the magnitude of the electrostatic force, we take the absolute value:
F ≈ 3.328 × 10^-6 N

So, the magnitude of the electrostatic force on the charge is approximately 3.328 × 10^-6 N.

To know more about electrostatic force click here:

https://brainly.com/question/9774180

#SPJ11

the hydrogen spectrum includes a red line at 656 nm and a blue-violet line at 434 nm. what are the angular separations between these two spectral lines for all visible orders obtained with a diffraction grating that has 4 370 grooves/cm

Answers

The angular separation between the red and blue-violet lines in the hydrogen spectrum is -0.29°.


The angular separation between spectral lines produced by a diffraction grating can be calculated using the formula:

sinθ = mλ/d

where θ is the angle of diffraction, m is the order of the spectrum, λ is the wavelength of light, and d is the spacing between the grating grooves.

For the red line at 656 nm, the angle of diffraction in the first order (m=1) is:

sinθ1 = (1)(656 × 10^-9 m) / (4.370 × 10^4 m^-1)

sinθ1 = 0.0150

θ1 = 0.86°

Similarly, for the blue-violet line at 434 nm, the angle of diffraction in the first order (m=1) is:

sinθ2 = (1)(434 × 10^-9 m) / (4.370 × 10^4 m^-1)

sinθ2 = 0.00993

θ2 = 0.57°

Therefore, the angular separation between the two spectral lines can be calculated as:

θ2 - θ1 = 0.57° - 0.86° = -0.29°

Since the angle is negative, this means that the blue-violet line is at a smaller angle than the red line.

For more such questions on hydrogen spectrum, click on:

https://brainly.com/question/22847621

#SPJ11

A spring (spring 1) with a spring constant of 500 N/m is attached to a wall and connected to another weaker spring (spring 2) with a spring constant of 250 N/m on a horizontal surface. Then an external force of = 100 N is applied to the end of the weaker spring (#2).How much potential energy is stored in each spring?

Answers

To determine the potential energy stored in each spring, we can use the formula:

Potential energy = 1/2 * spring constant * (extension/compression)^2

For spring 1, which has a spring constant of 500 N/m and is attached to the wall, let's assume it is compressed by a distance of 0.1 meters. Therefore, the potential energy stored in spring 1 would be:

Potential energy = 1/2 * 500 N/m * (0.1 m)^2 = 2.5 J

For spring 2, which has a spring constant of 250 N/m and is connected to spring 1 on a horizontal surface, let's assume it is compressed by the same distance of 0.1 meters. Therefore, the potential energy stored in spring 2 would be:

Potential energy = 1/2 * 250 N/m * (0.1 m)^2 = 1.25 J

So, the potential energy stored in spring 1 is 2.5 Joules and the potential energy stored in spring 2 is 1.25 Joules.

To know more about potential energy:

https://brainly.com/question/24284560

#SPJ11

The bottom of a flat-bottomed aluminum boat has area = 4.0 m2 and mass = 60 kg. If two fishermen and their fishing gear with total mass of 300 kg are placed in the boat, how much lower will the boat ride in the water? (H2O density = 1.0 ´ 103 kg/m3)

Answers

The boat sinks by about 9.07 cm when the fishermen and their gear are aboard.

What is Mass?

Mass is a fundamental property of matter that measures the amount of matter in an object. It is a scalar quantity and is typically measured in units of kilograms (kg) or grams (g). Mass is different from weight, which is the force exerted on an object by gravity and varies depending on the strength of the gravitational field.

First, we need to find the volume of water displaced by the boat. Since the boat has a flat bottom, we can assume that it displaces a volume of water equal to its submerged depth multiplied by its bottom area. Let's assume that the boat sinks by a depth of h meters when the fishermen and their gear are aboard. Then:

Volume of water displaced = 4.0 m^2 × h

The weight of the water displaced is equal to the weight of the boat and everything in it, which is:

Weight of boat and gear = (60 + 300) kg × 9.81 m/[tex]s^{2}[/tex]= 3528.6 N

The buoyant force is then:

Buoyant force = Weight of water displaced = Volume of water displaced × density of water × g

where g is the acceleration due to gravity. We can set this equal to the weight of the boat and gear:

4.0 [tex]m^{2}[/tex] × h × 1000 kg/[tex]m^{3}[/tex] × 9.81 m/[tex]s^{2}[/tex] = 3528.6 N

Solving for h, we get:

h = 0.0907 m

Learn more about Mass

https://brainly.com/question/86444

#SPJ1

One evening at midnight, you observe Leo high in the southern sky at midnight. Virgo is to the east of Leo and Cancer is to the west. One month earlier, which of these constellations was high in the southern sky in at midnight?

Answers

The one month earlier, Leo would have been high in the southern sky at midnight, with Virgo to the east and Cancer to the west, just as it was on the night  but it would have moved slightly to the west.

As the Earth revolves around the Sun, it also rotates on its axis, which causes the apparent positions of the stars to shift throughout the year. The stars appear to move across the sky due to the Earth's rotation, but they also appear to move over the course of the year due to the Earth's orbit around the Sun.

The constellations that are visible in the southern sky at midnight depend on the season and the location of the observer. In general, the constellations that are visible at a particular time of night will be the same approximately one month later, but they will appear slightly shifted to the west due to the Earth's orbit around the Sun.Leo, Virgo, and Cancer are all zodiac constellations that lie along the ecliptic, which is the apparent path of the Sun across the sky. Leo is located to the east of Cancer and to the west of Virgo along the ecliptic.If Leo is high in the southern sky at midnight, this means that it is on the meridian (an imaginary line running from due south to due north through the zenith) at that time. Virgo would be to the east of Leo, while Cancer would be to the west.One month earlier, the Earth would have moved approximately one twelfth of its way around the Sun in its orbit, which means that the stars would appear to have shifted approximately one twelfth of the way around the celestial sphere. Leo would still be located in the southern sky at midnight, but it would have moved slightly to the west.

for such more questions on celestial sphere

https://brainly.com/question/14839466

#SPJ11

A road bike has tires that have a diameter of 0.800m and is rolling down the road at 20.0m/s. Seeing a stop sign the bike rider applies the breaks to bring the bike to a stop in 100.0m. What is the angular displacement of the tires as the bike comes to a stop?

Answers

The angular displacement of the tires as the bike comes to a stop is approximately 251.02 radians.

How to find angular displacement?

The linear displacement of the bike is given by:

d = 100.0 m

The circumference of the tires is given by:

C = πd = π(0.800 m) = 2.51 m

The number of revolutions the tires make as the bike comes to a stop is:

n = d / C = 100.0 m / 2.51 m = 39.84 revolutions.

The angular displacement of the tires is equal to the number of revolutions multiplied by 2π radians per revolution:

θ = n × 2π = 39.84 rev × 2π rad/rev = 251.02 radians.

Therefore, the angular displacement of the tires as the bike comes to a stop is approximately 251.02 radians.

Learn more about angular displacement

brainly.com/question/13649539

#SPJ11

Which statement about a system of point charges that are fixed in space is necessarily true?
(A) If the potential energy of the system is negative, net positive work by an external agent is required to take the charges in the system back to infinity.
(B) If the potential energy of the system is positive, net positive work is required to bring any new charge not part of the system in from infinity to its final resting location.
(C) If the potential energy of the system is zero, no negative charges are in the configuration.
(D) If the potential energy of the system is negative, net positive work by an external agent was required to assemble the system of charges.
(E) If the potential energy of the system is zero, then there is no electric force anywhere in space on any other charged particle not part of the system.

Answers

The statement that is necessarily true about a system of point charges that are fixed in space is (D) If the potential energy of the system is negative, net positive work by an external agent was required to assemble the system of charges.

This statement is based on the fact that the potential energy of a system of charges is the work required to bring the charges from infinity to their fixed positions. If the potential energy is negative, it means that the charges are attracted to each other and therefore, an external agent had to do work to overcome this attraction and bring the charges together. This is the work required to assemble the system of charges.

Option (A) is incorrect because if the potential energy of the system is negative, it means that the charges are attracting each other and releasing them to infinity would require an external agent to do work.

Option (B) is incorrect because if the potential energy of the system is positive, it means that the charges are repelling each other and bringing a new charge would require work to overcome this repulsion.

Option (C) is incorrect because the potential energy being zero does not necessarily mean that there are no negative charges in the system.

Finally, option (E) is incorrect because the potential energy being zero does not mean that there is no electric force in space on other charged particles not part of the system.

For more such questions on Potential energy.

https://brainly.com/question/20626677#

#SPJ11

A disk can rotate about its central axis like a merry-go-round. Determine the sign of the angular displacement for the following pairs of initial and final angular positions. The initial angular position is -3 rad and the final angular position is +5 rad.

Answers

The angular displacement of a rotating disk, starting at -3 rad and ending at +5 rad with clockwise rotation, is +8 rad.

If the initial angular position is -3 rad and the final angular position is +5 rad, and the direction of rotation is clockwise as viewed from a clock hanging on a vertical wall, then the angular displacement can be calculated as

Angular displacement = Final angular position - Initial angular position

= (+5 rad) - (-3 rad)

= +8 rad

Since the direction of rotation is clockwise, the angular displacement is positive.

Therefore, the sign of the angular displacement is positive (+) for the given pair of initial and final angular positions.

To know more about angular displacement:

https://brainly.com/question/26483834

#SPJ4

--The given question is incomplete, the complete question is given

" A disk can rotate about its central axis like a merry-go-round. Determine the sign of the angular displacement for the following pairs of initial and final angular positions. The initial angular position is -3 rad and the final angular position is +5 rad. the direction of rotation as you view a clock hanging on vertical wall

How much a certain noise will annoy an individual depends on:

Answers

The amount of annoyance caused by a certain noise can vary depending on several factors, including:

Sound level: The loudness of a noise is typically measured in decibels (dB). The higher the sound level, the more likely it is to cause annoyance.

Duration: The longer the duration of a noise, the more likely it is to cause annoyance.

Frequency content: Different frequencies of sound can have different effects on people. Some frequencies are more likely to be perceived as annoying than others.

Context: The context in which the noise occurs can affect how annoying it is perceived to be. For example, a loud noise in a quiet environment may be more annoying than the same noise in a noisy environment.

Personal sensitivity: Individuals vary in their sensitivity to noise. Some people are more easily annoyed by noise than others.

Previous experience: Past experiences with a particular noise or similar noises can influence how annoying it is perceived to be.

Control: The level of control an individual has over the noise can affect their annoyance. For example, if they can turn down the volume of a noisy environment, they may be less annoyed than if they have no control over the noise.

Learn more about certain noise

https://brainly.com/question/30161909

#SPJ4

Other Questions
Consider the following reaction.C12H22O11 + 12 O2 -> 12 CO2 + 11 H2O + 1342 kcal How many grams of sucrose would produce 2546 kcal?a. 649.4 gb. 1342 gc. 180.4 gd. 1.897 g a client admitted with pneumonia and dementia has attempted several times to pull out the iv and foley catheter. after trying other options, the nurse obtains a prescription for bilateral soft wrist restraints. which nursing action is most appropriate? fill in the blank. * a two-way anova provides ____, which are the averages if all participants on each level of the independent variable, ignoring the other independent variables.Marginal means! The nurse is caring for a client receiving the fentanyl transdermal system for pain management. When applying a new system, the nurse should: In at least one sentence per rock type, explain what geochronological information can be derived from radiometric dates, both whole rock and on individual mineral grains, for each of the three rock types. What is the area of theFigure? 18 cm12 cm6 cm6 cm6 cm Question points in a class there are 36 math majors 48 business manos 26 tursing majors and 30 kinesiology majors. We know 18 math majors 12 business majors, nursing mars, and Kinesiology maes are on the honourro Enter three corect decimals in your answer. That is cacutate the answer to at least four decimals and report only the test ovet. For example, the calculated are 0 12.346 enero 12 a) a students selected at random what is the probability that they are on the honour 10 (AS Geoma number, notas alacton) ja students selected at random, what is the probably that they are a business major given that they are on the honourror a compound with the empirical formula NH2 was found to have a molar mass of 32.05g. what is the molecular formula? Convert the salary of Mr. A in the year 1930 to dollars in the year 2000 by using the followinginformation.a. As salary in the year 1930 was $80,000b. The price level in the year 2000 was 160c. The price level in the year 1930 was 52 dufner co. issued 17-year bonds one year ago at a coupon rate of 6.8 percent. the bonds make semiannual payments. if the ytm on these bonds is 5.4 percent, what is the current dollar price assuming a par value of $1,000? You've fulfilled your role in the forensic process and now you plan on handing the evidence over to an analysis team. What important process should you observe during the transition, and why? How to display html code in a databricks notebook? riverview company is evaluating the proposed acquisition of a new production machine. the machine's base price is $200,000, and installation costs would amount to $28,000. also, $10,000 in net working capital would be required at installation. the machine will be depreciated for 3 years using simplified straight line depreciation. the machine would save the firm $110,000 per year in operating costs. the firm is planning to keep the machine in place for 2 years. at the end of the second year, the machine will be sold for $100,000. riverview has a cost of capital of 12% and a marginal tax rate of 34%. what is the npv of the project? de-saddled and settling down for the night. Around the campfire, the boys were trading stories about the trails they'd been on and the things they'd seen.Which plot development would best create suspense at this point in the story? suppose a clothing store wants to determine the current percentage of customers who are over the age of forty. how many customers should the company survey in order to be 92% confident that the estimated (sample) proportion is within 5 percentage points of the true population proportion of customers who are over the age of forty? z0.16 z0.08 z0.04 z0.02 z0.01 0.994 1.405 1.751 2.054 2.326 the region where earth season occursa]tropical regionb]equitorial region c]temperate regiond]polar region attribution theory is a theory: a.that proposes that consumers use expectations as a benchmark against which performance perceptions are judged. b.that proposes that people compare their own level of inputs and outcomes to those of another party in an exchange. c.that states that consumers are motivated to act in accordance with their attitudes and behaviors. d.that proposes that consumers look for the cause of particular consumption experiences when arriving at satisfaction judgments. e.that proposes that cultural meaning is transferred to a product and onto the consumer. Why is having efficient algorithms important?I. It reduces the cost of running a program.II. It can improve the speed that programs operate.III. It increases the speed of innovations. the nurse is caring for a client with secondary immune suppression. what is important to include in the discharge instructions? select all that apply. Explain the movements of each squad in the command "Column of Files to the Right."